Stay ahead of learning milestones! Enroll in a class over the summer!

G
Topic
First Poster
Last Poster
k a May Highlights and 2025 AoPS Online Class Information
jlacosta   0
May 1, 2025
May is an exciting month! National MATHCOUNTS is the second week of May in Washington D.C. and our Founder, Richard Rusczyk will be presenting a seminar, Preparing Strong Math Students for College and Careers, on May 11th.

Are you interested in working towards MATHCOUNTS and don’t know where to start? We have you covered! If you have taken Prealgebra, then you are ready for MATHCOUNTS/AMC 8 Basics. Already aiming for State or National MATHCOUNTS and harder AMC 8 problems? Then our MATHCOUNTS/AMC 8 Advanced course is for you.

Summer camps are starting next month at the Virtual Campus in math and language arts that are 2 - to 4 - weeks in duration. Spaces are still available - don’t miss your chance to have an enriching summer experience. There are middle and high school competition math camps as well as Math Beasts camps that review key topics coupled with fun explorations covering areas such as graph theory (Math Beasts Camp 6), cryptography (Math Beasts Camp 7-8), and topology (Math Beasts Camp 8-9)!

Be sure to mark your calendars for the following upcoming events:
[list][*]May 9th, 4:30pm PT/7:30pm ET, Casework 2: Overwhelming Evidence — A Text Adventure, a game where participants will work together to navigate the map, solve puzzles, and win! All are welcome.
[*]May 19th, 4:30pm PT/7:30pm ET, What's Next After Beast Academy?, designed for students finishing Beast Academy and ready for Prealgebra 1.
[*]May 20th, 4:00pm PT/7:00pm ET, Mathcamp 2025 Qualifying Quiz Part 1 Math Jam, Problems 1 to 4, join the Canada/USA Mathcamp staff for this exciting Math Jam, where they discuss solutions to Problems 1 to 4 of the 2025 Mathcamp Qualifying Quiz!
[*]May 21st, 4:00pm PT/7:00pm ET, Mathcamp 2025 Qualifying Quiz Part 2 Math Jam, Problems 5 and 6, Canada/USA Mathcamp staff will discuss solutions to Problems 5 and 6 of the 2025 Mathcamp Qualifying Quiz![/list]
Our full course list for upcoming classes is below:
All classes run 7:30pm-8:45pm ET/4:30pm - 5:45pm PT unless otherwise noted.

Introductory: Grades 5-10

Prealgebra 1 Self-Paced

Prealgebra 1
Tuesday, May 13 - Aug 26
Thursday, May 29 - Sep 11
Sunday, Jun 15 - Oct 12
Monday, Jun 30 - Oct 20
Wednesday, Jul 16 - Oct 29

Prealgebra 2 Self-Paced

Prealgebra 2
Wednesday, May 7 - Aug 20
Monday, Jun 2 - Sep 22
Sunday, Jun 29 - Oct 26
Friday, Jul 25 - Nov 21

Introduction to Algebra A Self-Paced

Introduction to Algebra A
Sunday, May 11 - Sep 14 (1:00 - 2:30 pm ET/10:00 - 11:30 am PT)
Wednesday, May 14 - Aug 27
Friday, May 30 - Sep 26
Monday, Jun 2 - Sep 22
Sunday, Jun 15 - Oct 12
Thursday, Jun 26 - Oct 9
Tuesday, Jul 15 - Oct 28

Introduction to Counting & Probability Self-Paced

Introduction to Counting & Probability
Thursday, May 15 - Jul 31
Sunday, Jun 1 - Aug 24
Thursday, Jun 12 - Aug 28
Wednesday, Jul 9 - Sep 24
Sunday, Jul 27 - Oct 19

Introduction to Number Theory
Friday, May 9 - Aug 1
Wednesday, May 21 - Aug 6
Monday, Jun 9 - Aug 25
Sunday, Jun 15 - Sep 14
Tuesday, Jul 15 - Sep 30

Introduction to Algebra B Self-Paced

Introduction to Algebra B
Tuesday, May 6 - Aug 19
Wednesday, Jun 4 - Sep 17
Sunday, Jun 22 - Oct 19
Friday, Jul 18 - Nov 14

Introduction to Geometry
Sunday, May 11 - Nov 9
Tuesday, May 20 - Oct 28
Monday, Jun 16 - Dec 8
Friday, Jun 20 - Jan 9
Sunday, Jun 29 - Jan 11
Monday, Jul 14 - Jan 19

Paradoxes and Infinity
Mon, Tue, Wed, & Thurs, Jul 14 - Jul 16 (meets every day of the week!)

Intermediate: Grades 8-12

Intermediate Algebra
Sunday, Jun 1 - Nov 23
Tuesday, Jun 10 - Nov 18
Wednesday, Jun 25 - Dec 10
Sunday, Jul 13 - Jan 18
Thursday, Jul 24 - Jan 22

Intermediate Counting & Probability
Wednesday, May 21 - Sep 17
Sunday, Jun 22 - Nov 2

Intermediate Number Theory
Sunday, Jun 1 - Aug 24
Wednesday, Jun 18 - Sep 3

Precalculus
Friday, May 16 - Oct 24
Sunday, Jun 1 - Nov 9
Monday, Jun 30 - Dec 8

Advanced: Grades 9-12

Olympiad Geometry
Tuesday, Jun 10 - Aug 26

Calculus
Tuesday, May 27 - Nov 11
Wednesday, Jun 25 - Dec 17

Group Theory
Thursday, Jun 12 - Sep 11

Contest Preparation: Grades 6-12

MATHCOUNTS/AMC 8 Basics
Friday, May 23 - Aug 15
Monday, Jun 2 - Aug 18
Thursday, Jun 12 - Aug 28
Sunday, Jun 22 - Sep 21
Tues & Thurs, Jul 8 - Aug 14 (meets twice a week!)

MATHCOUNTS/AMC 8 Advanced
Sunday, May 11 - Aug 10
Tuesday, May 27 - Aug 12
Wednesday, Jun 11 - Aug 27
Sunday, Jun 22 - Sep 21
Tues & Thurs, Jul 8 - Aug 14 (meets twice a week!)

AMC 10 Problem Series
Friday, May 9 - Aug 1
Sunday, Jun 1 - Aug 24
Thursday, Jun 12 - Aug 28
Tuesday, Jun 17 - Sep 2
Sunday, Jun 22 - Sep 21 (1:00 - 2:30 pm ET/10:00 - 11:30 am PT)
Monday, Jun 23 - Sep 15
Tues & Thurs, Jul 8 - Aug 14 (meets twice a week!)

AMC 10 Final Fives
Sunday, May 11 - Jun 8
Tuesday, May 27 - Jun 17
Monday, Jun 30 - Jul 21

AMC 12 Problem Series
Tuesday, May 27 - Aug 12
Thursday, Jun 12 - Aug 28
Sunday, Jun 22 - Sep 21
Wednesday, Aug 6 - Oct 22

AMC 12 Final Fives
Sunday, May 18 - Jun 15

AIME Problem Series A
Thursday, May 22 - Jul 31

AIME Problem Series B
Sunday, Jun 22 - Sep 21

F=ma Problem Series
Wednesday, Jun 11 - Aug 27

WOOT Programs
Visit the pages linked for full schedule details for each of these programs!


MathWOOT Level 1
MathWOOT Level 2
ChemWOOT
CodeWOOT
PhysicsWOOT

Programming

Introduction to Programming with Python
Thursday, May 22 - Aug 7
Sunday, Jun 15 - Sep 14 (1:00 - 2:30 pm ET/10:00 - 11:30 am PT)
Tuesday, Jun 17 - Sep 2
Monday, Jun 30 - Sep 22

Intermediate Programming with Python
Sunday, Jun 1 - Aug 24
Monday, Jun 30 - Sep 22

USACO Bronze Problem Series
Tuesday, May 13 - Jul 29
Sunday, Jun 22 - Sep 1

Physics

Introduction to Physics
Wednesday, May 21 - Aug 6
Sunday, Jun 15 - Sep 14
Monday, Jun 23 - Sep 15

Physics 1: Mechanics
Thursday, May 22 - Oct 30
Monday, Jun 23 - Dec 15

Relativity
Mon, Tue, Wed & Thurs, Jun 23 - Jun 26 (meets every day of the week!)
0 replies
jlacosta
May 1, 2025
0 replies
orthocenter on sus circle
DVDTSB   2
N 10 minutes ago by Diamond-jumper76
Source: Romania TST 2025 Day 2 P1
Let \( ABC \) be an acute triangle with \( AB < AC \), and let \( O \) be the center of its circumcircle. Let \( A' \) be the reflection of \( A \) with respect to \( BC \). The line through \( O \) parallel to \( BC \) intersects \( AC \) at \( F \), and the tangent at \( F \) to the circle \( \odot(BFC) \) intersects the line through \( A' \) parallel to \( BC \) at point \( M \). Let \( K \) be a point on the ray \( AB \), starting at \( A \), such that \( AK = 4AB \).
Show that the orthocenter of triangle \( ABC \) lies on the circle with diameter \( KM \).

Proposed by Radu Lecoiu

2 replies
DVDTSB
Yesterday at 12:18 PM
Diamond-jumper76
10 minutes ago
problem 5
termas   74
N 22 minutes ago by maromex
Source: IMO 2016
The equation
$$(x-1)(x-2)\cdots(x-2016)=(x-1)(x-2)\cdots (x-2016)$$is written on the board, with $2016$ linear factors on each side. What is the least possible value of $k$ for which it is possible to erase exactly $k$ of these $4032$ linear factors so that at least one factor remains on each side and the resulting equation has no real solutions?
74 replies
termas
Jul 12, 2016
maromex
22 minutes ago
Inspired by nhathhuyyp5c
sqing   2
N 25 minutes ago by lbh_qys
Source: Own
Let $ x,y>0, x+2y- 3xy\leq 4. $Prove that
$$ \frac{1}{x^2} + 2y^2 + 3x + \frac{4}{y} \geq 3\left(2+\sqrt[3]{\frac{9}{4} }\right)$$
2 replies
sqing
an hour ago
lbh_qys
25 minutes ago
I think I know why this problem was rejected by IMO PSC several times...
mshtand1   1
N 42 minutes ago by sarjinius
Source: Ukrainian Mathematical Olympiad 2025. Day 2, Problem 11.8
Exactly $102$ country leaders arrived at the IMO. At the final session, the IMO chairperson wants to introduce some changes to the regulations, which the leaders must approve. To pass the changes, the chairperson must gather at least \(\frac{2}{3}\) of the votes "FOR" out of the total number of leaders. Some leaders do not attend such meetings, and it is known that there will be exactly $81$ leaders present. The chairperson must seat them in a square-shaped conference hall of size \(9 \times 9\), where each leader will be seated in a designated \(1 \times 1\) cell. It is known that exactly $28$ of these $81$ leaders will surely support the chairperson, i.e., they will always vote "FOR." All others will vote as follows: At the last second of voting, they will look at how their neighbors voted up to that moment — neighbors are defined as leaders seated in adjacent cells \(1 \times 1\) (sharing a side). If the majority of neighbors voted "FOR," they will also vote "FOR." If there is no such majority, they will vote "AGAINST." For example, a leader seated in a corner of the hall has exactly $2$ neighbors and will vote "FOR" only if both of their neighbors voted "FOR."

(a) Can the IMO chairperson arrange their $28$ supporters so that they vote "FOR" in the first second of voting and thereby secure a "FOR" vote from at least \(\frac{2}{3}\) of all $102$ leaders?

(b) What is the maximum number of "FOR" votes the chairperson can obtain by seating their 28 supporters appropriately?

Proposed by Bogdan Rublov
1 reply
mshtand1
Mar 14, 2025
sarjinius
42 minutes ago
No more topics!
APMO 2012 #3
syk0526   30
N Apr 23, 2025 by EVKV
Source: APMO 2012 #3
Determine all the pairs $ (p , n )$ of a prime number $ p$ and a positive integer $ n$ for which $ \frac{ n^p + 1 }{p^n + 1} $ is an integer.
30 replies
syk0526
Apr 2, 2012
EVKV
Apr 23, 2025
APMO 2012 #3
G H J
Source: APMO 2012 #3
The post below has been deleted. Click to close.
This post has been deleted. Click here to see post.
syk0526
202 posts
#1 • 5 Y
Y by Davi-8191, chessgocube, SADAT, Adventure10, Mango247
Determine all the pairs $ (p , n )$ of a prime number $ p$ and a positive integer $ n$ for which $ \frac{ n^p + 1 }{p^n + 1} $ is an integer.
Z K Y
The post below has been deleted. Click to close.
This post has been deleted. Click here to see post.
dinoboy
2903 posts
#2 • 4 Y
Y by Binomial-theorem, chessgocube, Adventure10, Mango247
Ah APMO problems have been posted! :D A bit easy for APMO though :(

First remark that if $p < n$, then $p^n > n^p$ so $p^n + 1 > n^p + 1$ and the problem obviously doesn't hold. Hence $p \ge n$.
Consider a prime power $z^k|(p+1)$. Unless $n \equiv -1 \pmod{z^k}$ we have $\text{ord}_{z^k}(n) = 2p$. But that's obviously false as $2p > z^k$, hence $n \equiv -1 \pmod{z^k}$ or all prime powers which divides $(p+1)$. But this implies $n \equiv -1 \pmod{p+1}$, and hence as $p \ge n$ the only solutions are when $n=p$ which obviously work. Hence all solutions are $(p,p)$ for $p$ a prime number.

EDIT : Oops my inequality fails when $p=2$, so $(2,4)$ is also a solution. Here is a proof $a^b > b^a$ when $a > 2$ and $a < b$ to make sure I don't mess up more.
It suffices to show $a^{b/a} > b$. When $a > e \approx 2.718$ we know $a^z > za$ for $z > 1$ so the result follows. To prove this, consider the function $f(x) = a^x - ax$. $f'(x) = \ln a a^x - a$. Remark that $f'(x) \ge 0$ for $x \ge 0$ and the result follows as $f(1) = 0$.

Now for $p=2$, $2^n > n^2$ for $n > 4$ so it follows the only solutions are $(p,p)$ and $(2,4)$.
This post has been edited 1 time. Last edited by dinoboy, Apr 2, 2012, 4:39 PM
Z K Y
The post below has been deleted. Click to close.
This post has been deleted. Click here to see post.
mlm95
245 posts
#3 • 6 Y
Y by perfect_square, ---Fermat---, ZHEKSHEN, chessgocube, Adventure10, Mango247
I'm not agree with you $p=2$ and $n=4$ is an answer notice that $2^{4}=4^{2}$
Z K Y
The post below has been deleted. Click to close.
This post has been deleted. Click here to see post.
WakeUp
1347 posts
#4 • 8 Y
Y by dizzy, Binomial-theorem, AdiletR, Orkhan-Ashraf_2002, Vietjung, chessgocube, Adventure10, Mango247
My solution is slightly different from that of dinoboy's:

If $p=2$ then $n=2$ and by the inequality $2^n>n^2$ for $n>4$, we get that $n=2$ or $n=4$ work.

Otherwise $p\ge 3$ so $p$ is odd, meaning $n$ must be also odd since $2|n^p+1$. Hence $p^n+1$ can be factorised as $(p+1)(p^{n-1}-p^{n-2}+\ldots +p^2-p+1)$. Thus $p+1|n^p+1$. Let $q$ be a prime factor of $p+1$. Suppose $q$ is odd. Now $q|n^p+1$ so $n^{p}=-1\pmod{q}$. Since $q$ is odd, this means $\text{ord}_q(n)\in\{2,2p\}$.

If $\text{ord}_q(n)=2$ then $n^2=1\pmod{q}$. If $n=1\pmod{q}$ then $1=n^p=(-1)^p=-1\pmod{q}$ but $q\nmid 2$, so this is impossible. Hence $n=-1\pmod{q}$ and so $q|n+1$.

If $\text{ord}_q(n)=2p$ then $2p|q-1$ (note that $n^{q-1}=1\pmod{q}$ by FLT) but also $q|p+1$. These divisibilities imply that either $q=1$ or $2p\le p$, both of which are of course contradictory.

So if $q\not= 2$ then $q|n+1$. If $q=2$ then clearly $q|n+1$ as $n$ is odd.

Now suppose for some prime divisor $r$ of $p+1$, we have that $v_r(p+1)>v_r(n+1)$ (we denote the number $k$ such that $a^k||b$ as $v_a(b)$). Therefore since $p+1|n^p+1=(n+1)(n^{p-1}-n^{p-2}+\ldots +n^2-n+1)$, we have that $r|n^{p-1}-n^{p-2}+\ldots +n^2-n+1$. But $n=-1\pmod{r}$ and $p$ is odd, implying that $0=n^{p-1}-n^{p-2}+\ldots +n^2-n+1=1+1+\ldots +1+1+1=p\pmod{r}$. Therefore $r|p$, impossible in view of the fact that $r|p+1$. Hence $v_r(p+1)\le v_r(n+1)$ for all prime divisors $r$ of $p+1$. Therefore $p+1|n+1\implies p\le n$. As dinoboy has already noted, we have must $p\ge n$, so overall $p=n$ giving all solutions to be $(p,p)$ for all primes $p$ and $(2,4)$.
Z K Y
The post below has been deleted. Click to close.
This post has been deleted. Click here to see post.
yunxiu
571 posts
#5 • 12 Y
Y by Vietjung, golema11, puntre, ZHEKSHEN, chessgocube, Mathlover_1, Adventure10, Mango247, and 4 other users
An other solution:
If $p = 2$, then ${n^2} + 1 \geqslant {2^n} + 1$, so we have $n \leqslant 4$, it is easy to see $n = 2,4$.
Now we suppose $p$ is an odd prime, so $n$ is odd, because ${n^p} + 1 \geqslant {p^n} + 1 > 3$, so $n \geqslant 3$.
Because $f(x) = {x^{1/x}}$ is decrease for $x \geqslant e$, from ${n^{1/n}} \geqslant {p^{1/p}}$ we have $n \leqslant p$.
Because $p + 1\left| {{p^n} + 1} \right.$, ${n^p} + 1 \equiv 0 \equiv {p^p} + 1(\bmod p + 1)$, so ${n^p} \equiv {p^p}(\bmod p + 1)$. By $Euler$ theorem ${n^{\varphi (p + 1)}} \equiv 1 \equiv {p^{\varphi (p + 1)}}(\bmod p + 1)$, so ${n^{(p,\varphi (p + 1))}} \equiv {p^{(p,\varphi (p + 1))}}(\bmod p + 1)$, hence $n \equiv p(\bmod p + 1)$, because $1 \leqslant n \leqslant p$, we have $n = p$.
Above all, $(p,n) = (p,p),(2,4)$ are the all solutions.
Z K Y
The post below has been deleted. Click to close.
This post has been deleted. Click here to see post.
AwesomeToad
4535 posts
#6 • 3 Y
Y by Ibraheem, Adventure10, Mango247
Why does $ {n^{(p,\varphi (p+1))}}\equiv{p^{(p,\varphi (p+1))}}(\bmod p+1) $ imply $n\equiv p \mod p+1$?
Z K Y
The post below has been deleted. Click to close.
This post has been deleted. Click here to see post.
yunxiu
571 posts
#7 • 4 Y
Y by AwesomeToad, ZHEKSHEN, Adventure10, Mango247
AwesomeToad wrote:
Why does $ {n^{(p,\varphi (p+1))}}\equiv{p^{(p,\varphi (p+1))}}(\bmod p+1) $ imply $n\equiv p \mod p+1$?

Because $p$ is a odd prime, $p + 1$ is even, so every prime factor of $\varphi (p + 1)$ is less then $\frac{{p + 1}}{2} < p$, hence $\left( {p,\varphi (p + 1)} \right) = 1$.
Z K Y
The post below has been deleted. Click to close.
This post has been deleted. Click here to see post.
sicilianfan
944 posts
#8 • 2 Y
Y by Adventure10, Mango247
Can someone tell me if this solution is valid or not? I'm not really to sure about it so I would appreciate any input. Thanks in advance. (Also, I wasn't sure whether to make a new thread or to revive this old one, so I apologize if I have done the wrong thing)

my solution
Z K Y
The post below has been deleted. Click to close.
This post has been deleted. Click here to see post.
Konigsberg
2229 posts
#9 • 2 Y
Y by Adventure10, Mango247
@sicilianfan: Your solution is correct. Nice argument regarding how much each quantity increases! And also the fact that $n \cong -1 (mod p+1)$.
Z K Y
The post below has been deleted. Click to close.
This post has been deleted. Click here to see post.
sicilianfan
944 posts
#10 • 2 Y
Y by Adventure10, Mango247
Thanks for the feedback, but there actually was a problem with my solution (of which I was informed a while ago): I assumed that just because $\text{ord}_{p+1}(n)=2$ that $n \equiv -1 \pmod{p+1}$ which is in fact only true if $p+1$ has a primitive root (this is what I overlooked), which is certainly not a valid assumption. It can be fixed by taking mod $q$ where $q$ is a prime that divides $p+1$ instead, and patching from there.
Z K Y
The post below has been deleted. Click to close.
This post has been deleted. Click here to see post.
liberator
95 posts
#11 • 7 Y
Y by Ferid.---., jlammy, rightways, Adventure10, Mango247, farhad.fritl, and 1 other user
If $p=2$, then $n^2 + 1 \geq 2^n + 1 \implies n \leq 4$. Easily, we can see $(p,n) = (2,2); (2,4)$ are the only solution in this case.

Suppose now that $p \geq 3$ is an odd prime. Since $n^p + 1 \geq p^n + 1$, we have $n \geq 3$. Now $n^p \geq p^n \implies n^{\frac{1}{n}} \geq p^{\frac{1}{p}} \implies n \leq p$ since $f(x) = x^{\frac{1}{x}}$ is a decreasing function for $x \geq e$.

$(p+1) \mid p^n + 1 \implies (p+1) \mid n^p + 1$. Hence $n^p \equiv -1 \pmod {p+1}$, so it follows that $\text{ord}_{p+1}(n) = 2$ or $2p$. The second option is absurd, since $2p > p+1 > \phi (p+1)$, so $\text{ord}_{p+1} (n) = 2$: i.e. $n^2 \equiv 1 \pmod {p+1}$. Now $-1 \equiv n^p \equiv n^{p-1}\cdot n \equiv n \pmod {p+1}$, so $p=n$.

Thus our only solutions are $\boxed{(p,n) = (p,p); (2,4)}$.
Z K Y
The post below has been deleted. Click to close.
This post has been deleted. Click here to see post.
vi1lat
189 posts
#12 • 2 Y
Y by Adventure10, Mango247
What is the mean $ord$??
Z K Y
The post below has been deleted. Click to close.
This post has been deleted. Click here to see post.
JackXD
151 posts
#13 • 2 Y
Y by Adventure10, Mango247
Let $p$ be an odd prime.

Then $p^n+1|n^p+1 \implies p^{\frac{1}{p}} \le n^{\frac{1}{n}} \implies n \le p$ because $f(x)=x^{\frac{1}{x}}$ is decreasing function for $x \ge e$.
It is easy to see that $n$ must be odd.Let $r$ be the order of $n$ modulo $p+1$.Then $p^n+1|n^p+1 \implies p+1|n^{2p}-1 \implies r|2p$.Clearly $r \le p$ and $r$ cannot be $p$.Also $r=1 \implies p+1|n^p-1 \implies p+1|2 $ which is absurd.So $r=2$ and $p+1|n+1$ which means $p \le n$.This along with the first line's inequality yeilds $n=p$.

For $p=2$ we have $2^n+1|n^2+1 \implies 2^n \le n^2 \implies n \le 4$.Checking we get solutions $n=2,4$.

Thus the solutions are $(p,n) \equiv (2,4),(p,p)$
Z K Y
The post below has been deleted. Click to close.
This post has been deleted. Click here to see post.
K.titu
321 posts
#14 • 2 Y
Y by Adventure10, Mango247
Hi dear mathlinkers
I have a question
What made you to think about $p+1$?
Z K Y
The post below has been deleted. Click to close.
This post has been deleted. Click here to see post.
uraharakisuke_hsgs
365 posts
#16 • 2 Y
Y by Adventure10, Mango247
Fisrt , if $p > 2$ then $n$ is odd . Let $q$ is a prime number of $p+1$ . Then , $q | n^p+1 \implies q | n^{2p}-1$ , and $q| n^{q-1}-1$ , and because $q < p$ then $ q | n^2 - 1$ , which implies $q | n+1$
And we have $v_q(p^n+1) = v_q (p+1) < v_q(n^p+1) = v_q(n+1) \implies p+1 | n+1 \implies p \leq n $
But we have $n^p > p^n \implies plog(n) \geq nlog(p) \implies \frac{log (n)}{n} \geq \frac{log (p) }{p} $
Let $f(x) = \frac{log x }{x}$ , then $f'(x) = \frac{1 - log x }{x^2}$ , then , for $x > 10$ , $f$ is decreasing . So , for $n , p > 10$ we have $ n \leq p$
Then $ n = p$
For the cases $n$ or $p < 10$ we can check that $(n,p) = (4,2)$
Z K Y
The post below has been deleted. Click to close.
This post has been deleted. Click here to see post.
ubermensch
820 posts
#17 • 1 Y
Y by Adventure10
Obviously we need $n^p \equiv -1 \pmod{p^n+1} => n^{2p}=1 \pmod{p^n+1}$.
Thus $O_{p^n +1}(n) \mid 2p$, but it doesn’t divide $p$- thus it must equal $2$ or $2p$.
First let’s deal with the easier case, where order is $2$. Here we can clearly see that as $n^2-1 \geq p^n+1$, which is obviously false.
Now, what’s left is the case where $O_{p^n+1}(n)=2p$.
In this case, we must again split into cases- before that, let’s consider some prime $q$ such that $v_q(p+1)=\alpha>0$- here notice that as $n \equiv 1\pmod{2}$ for odd primes, $q^{\alpha} \mid p^{n}+1$.
Clearly, we need $O_{q^\alpha}(n)= 2$ or $2p$, by similar logic- if $2p$, we get an immediate contradiction as $2p>p+1$.
Now let’s look at the case when $O_{q^{\alpha}}(n)=2$.
Now, as $(n+1)(n-1)= 0\pmod{q^{\alpha}}=> q^{\alpha} \mid n+1$ (almost forgot to consider $q=2$ separately, but in the end it doesn’t matter as $n$ is odd) for all such $q^{\alpha} \mid p+1$. Hence, in a rather elegant manner, we get $p+1 \mid n+1$, which of course kills the problem as $p \geq n => (p,p)$ is only solution for odd primes.

Now, you could manually check for $p=2$ and that’ll give you and extra solution $(2,4)$, hence concluding the problem with the solution set $(p,p),(2,4)$ for all primes $p$.

I felt this was actually quite a nice problem and a great fit for P3 (okay maybe a bit on the easier side, I suppose, but I’m not a very good judge), simply because if you don’t notice a few key points(at least if you solve it the way I did), the problem becomes hellish to solve- that makes me wonder, who proposed this?
This post has been edited 3 times. Last edited by ubermensch, Jul 27, 2019, 8:58 PM
Z K Y
The post below has been deleted. Click to close.
This post has been deleted. Click here to see post.
mira74
1010 posts
#18 • 12 Y
Y by tworigami, anish9876, GameMaster402, CantonMathGuy, giacomorizzo, kevinmathz, Kagebaka, Toinfinity, p_square, niyu, khina, IAmTheHazard
Made with some people.
Assume $p=2$, and deal with $p>2$ later.
Leaving $n \leq 4$, we have $n^2+1 < 2^n+1$, and there are no integers in $(0,1)$.
Looking at $n \leq 4$, we see $n=2 ,4$ work.
See that for $p$ odd, $p^n+1$ is even, so $n^p+1$ is even
Then, $n$ must be odd as well
Assume $n \neq p$, since we can see that $n=p$ works.
Realize that $p+1 \mid p^n+1$ from $n$ odd, so $p+1 \mid n^p+1$
Note that since any prime $q \mid p+1$ is $<p$
Abusing order, this implies $q \mid n^p+1 \Rightarrow q \mid n+1$.
Thus, $p+1 \mid n+1$, so $n>p$.
Inequalities kill the rest of the problem: we claim $n^p+1>p^n+1$.
Observe that $f(x)=\ln(\sqrt[x]{x})$ has $f'(x)=\frac{1-\ln(x)}{x^2}$, it decreases for $x>e$, so $\sqrt[n]{n} \leq \sqrt[p]{p}$, implying result.
Now, we are left with the final answer of $(2,4)$ and $(p,p)$.
YUH $\blacksquare$
This post has been edited 3 times. Last edited by mira74, Apr 6, 2020, 1:41 AM
Z K Y
The post below has been deleted. Click to close.
This post has been deleted. Click here to see post.
pad
1671 posts
#19
Y by
Assume $p$ is odd; we will deal with $p=2$ later. Then $p^n+1$ is even, so $n^p+1$ is even, so $n$ is odd. Suppose $q\mid p+1$ for a prime $q$. Then
\[ q\mid p+1 \implies q\mid p^n+1 \implies q\mid n^p+1\implies n^p\equiv -1 \pmod{q}, \ n^{2p} \equiv 1 \pmod{q}. \]Let the order of $n \pmod{q}$ be $x$. Then $x\nmid p, x\mid 2p$, so $x=2,2p$. We also have $x\mid q-1$; hence if $x=2p$ then $2p \mid q-1$, so $q \ge 2p$, contradicting $q \mid p+1$ by size. Hence $x=2$. So $n^2 \equiv 1\pmod{q}$, which means $q\mid (n-1)(n+1)$. We claim we actually have $q\mid n+1$.
  • If $q\mid n+1$, we have proved the claim.
  • If $q\mid n-1$, then $q\mid n^p-1$. But $q\mid p+1 \mid p^n+1 \mid n^p+1$, so we also have $q\mid 2$. So $q=2$. But since $n$ is odd, $2\mid n+1$, so this proves our claim.
Therefore, $q\mid p+1 \implies q\mid n+1$. Now, we use LTE. Note that $q\mid p+1, n+1$, so $q\nmid p,n$. We have
\begin{align*}
\nu_q(n^p+1) &= \nu_q(n+1) + \nu_q(p) = \nu_q(n+1)  \\
\nu_q(p^n+1) &= \nu_q(p+1) + \nu_q(n) = \nu_q(p+1).
\end{align*}But $p^n+1\mid n^p+1$, so therefore $\nu_q(n+1) \ge \nu_q(p+1)$. Since $q$ was defined to be any prime divisor of $p+1$, we conclude that $p+1\mid n+1$.

In particular, $p\le n$. Now, we use size. Since $p^n+1\mid n^p+1$, we know $p^n \le n^p$. So $n \ln p \le p \ln n$, i.e. $\tfrac{n}{\ln n} \le \tfrac{p}{\ln p}$. Let $f(x) = \tfrac{x}{\ln x}$. It is not hard to show that $f(x)$ is increasing for $x\ge 3$. But $x$ increased from $p$ to $n$, while $f(x)$ decreased from $p$ to $n$. Therefore, either $p<3$ or $n=p$. If $p<3$, then $p=2$, and only $(p,n)=(2,2),(2,4)$ works here. Finally, the solutions are $(p,n)=(p,p),(2,4)$.

Remarks
This post has been edited 3 times. Last edited by pad, May 23, 2020, 1:23 AM
Z K Y
The post below has been deleted. Click to close.
This post has been deleted. Click here to see post.
IAmTheHazard
5001 posts
#20 • 1 Y
Y by centslordm
The answer is $(2,4)$ and $(p,p)$, where $p$ is prime, only. These can be easily checked. For $p=2$, we can find that the only solutions are $n=2,4$, since anything larger results in $p^n+1>n^p+1$. Henceforth assume $p \geq 3$

Note that it is well-known that if $a>e>2$ and $b>a$, then $a^b>b^a$. Since we need $n^p+1 \geq p^n+1$ and $p \geq 3$, it follows that $b \leq a$. Now, since $p \geq 3$, $p^n+1$ is even and hence $n^p+1$ is even too, so $n$ is odd. As such, any prime $q \mid p+1$ must divide $p^n+1$. Let $Q=q^e \mid p+1$, where $q$ is a prime, $Q \neq 2$ and $e=v_q(p+1)$. Then we need $n^p \equiv -1 \pmod{Q}$. This is enough to imply that $\operatorname{ord}_Q(n) \mid 2p$. However, we also cannot have $\operatorname{ord}_Q(n) \mid p$, else $n^p \equiv 1 \pmod{Q}$. This narrows down the options to $\operatorname{ord}_Q(n)\in \{2,2p\}$. However, the latter is impossible for size reasons as $Q \leq p+1$, so $\operatorname{ord}_Q(n) \leq Q-1\leq p \leq 2p$. Hence we conclude that $\operatorname{ord}_Q(n)=2$ for all prime powers $Q \neq 2$ dividing $p+1$, implying $n \equiv -1 \pmod{Q}$. This also works if $Q=2$, in which case $n \equiv 1 \equiv -1 \pmod{Q}$. Hence by CRT, we have $n \equiv -1 \pmod{p+1}$. But we also have $b \leq a$, so $n=p$. Thus no other solutions exist. $\blacksquare$
Z K Y
The post below has been deleted. Click to close.
This post has been deleted. Click here to see post.
Sprites
478 posts
#22
Y by
syk0526 wrote:
Determine all the pairs $ (p , n )$ of a prime number $ p$ and a positive integer $ n$ for which $ \frac{ n^p + 1 }{p^n + 1} $ is an integer.


Assume that $n \neq p$ since when $n=p$ we obtain $(n,p)=(p,p)$ which works.
Henceforth assume that $p \ge n$ and $p \ge 2$ i.e when $p=2$ we obtain $(n,p)=(2,4)$ which works.
Clearly $n$ is odd let $\tilde{p}$ be a prime dividing $p+1$
Notice that $$\nu_{\tilde{p}}(p^n+1)=\nu_{\tilde{p}} (p+1)+\nu_{\tilde{p}} (n)$$i.e $$\tilde{p}|n^p+1 \implies ord_{\tilde{p}}(n)=\{1,2,2p\}$$The first and last cases are impossible hence $n^2+1 \equiv 0 \pmod {\tilde{p}}$ implying
$p+1 \equiv 1 \pmod 4 \implies 4|p$,contradiction so $(n,p)=(4,2),(p,p)$
This post has been edited 1 time. Last edited by Sprites, Oct 26, 2021, 6:00 AM
Z K Y
The post below has been deleted. Click to close.
This post has been deleted. Click here to see post.
lazizbek42
548 posts
#23
Y by
Lemma:if $a$ and $b$ positive integer numbers and $a \leq b \implies b^a \leq a^b$
Z K Y
The post below has been deleted. Click to close.
This post has been deleted. Click here to see post.
mijail
121 posts
#24
Y by
Funny problem! :D

Suppose that $p \neq 2$ because if we have $p=2$ then $n^2+1 \ge 2^n +1 \implies n \leq 4$ so we have the solutions: $(p,n): (2,2); (2,4)$

Claim: $p+1 \mid n+1$
Proof: First we have that $p^n+1$ is even so $n$ is odd then $p+1 \mid p^n+1\mid n^p+1$ so $p+1 \mid n^{2p} -1 \implies p+1 \mid n^{(2p,\phi(p+1))}-1$ but $\phi(p+1) \leq p-1$ so $(\phi(p+1),p)=1 \implies (2p,\phi(p+1)) \mid 2 \implies p+1 \mid n^2 -1 \implies p+1 \mid n+1$. (Because $p$ is odd) $\square$

Let $n=pu$ .Then we have that $p^n +1 \leq n^p +1 \implies p^pu \leq (pu)^p \implies p^{u-1} \leq u$ but if $u \ge 2$ we have that: $$\lfloor u \rfloor +1 > u \ge p^{u-1} \ge 3^{u-1}
 \ge 3^{\lfloor u \rfloor-1} \implies  \lfloor u \rfloor +1 \ > 3^{\lfloor u \rfloor-1} $$This is false if $u \ge 2$ so we have $n <2p$ and $p+1 \mid n+1$ so $p=n$. This concludes the problem. $\blacksquare$
Z K Y
The post below has been deleted. Click to close.
This post has been deleted. Click here to see post.
Jalil_Huseynov
439 posts
#25
Y by
I really enjoyed from this problem.
If $p=2$, we easily get $n=2$ or $n=4$. If $n=1$, we get there is no solution. If $n=2$ we get $p=2$. Now assume $p,n>2$. Since $p^n\leq n^p$ and $p,n>e$, we get $p\geq n$. $n=p$ is obviously solution. Now assume $p>n$. Since $p$ is odd, we get $n$ is odd also. So $p+1\mid n^p+1$. Consider odd prime divisor $q$ of $p+1$. Since $p>q-1$, we get $\gcd(2p,q-1)=2$. Since $n^{q-1}\equiv n^{2p}\equiv 1 \pmod{q}$, and $\gcd(2p,q-1)=2$, we get $n^2\equiv 1 \pmod{q}$. So $n^p+1\equiv (n^2)^{\frac{p-1}{2}} \cdot n+1\equiv n+1\equiv 0 \pmod{q}$. So $q\mid n+1$. Also we have $v_q({p^n+1})=v_q({p+1})+v_q({n})=v_q({p+1})\leq v_q({n^p+1})=v_q({n+1})+v_q({p})=v_q({n+1})$. So if $q$ is odd prime divisor of $p+1$ , then $v_q({p+1})\leq v_q({n+1})$. Now let's deal with $v_2$. Since $v_2({p+1})\leq v_2({n^p+1})=v_2({n+1})$, we get $v_2({p+1})\leq v_2({n+1})$. So $v_r({p+1})\leq v_r({n+1})$ for all prime number $r$. So we get $p\leq n$, which is contradiciton. So all solutions are $\boxed{n=p}$, $\boxed{p=2,n=4}$.
Z K Y
The post below has been deleted. Click to close.
This post has been deleted. Click here to see post.
ZNatox
67 posts
#26
Y by
If $p=2$, we get $n=2,4$.
Now suppose $p>2$, then $n$ is odd, and we can't have $n=1$, thus: $n \le p$.
$p+1 \mid p^n+1 \mid n^p+1 \mid n^{2p}-1$ which implies $O_{p+1}(n) \mid 2p$. If $O_{p+1}(n)$ is odd we get $O_{p+1}(n) \mid p$ clearly impossible. Thus: $O_{p+1}(n) \in \{2,2p\}$. Can we have: $O_{p+1}(n)=2p$? No, the reason being: $\phi(p+1)\le (p+1)<2p$. Thus: $O_{p+1}(n)=2$ which gives us that: $p+1 \mid (n-1)(n+1)$ or $\frac{p+1}{2^{v_2(p+1)}} \mid (n-1) × \frac{n+1}{2^{v_2(p+1)}}$, but $\frac{p+1}{2^{v_2(p+1)}}$ is odd, and therefore coprime with $n-1$, and by LTE $\frac{n+1}{2^{v_2(p+1)}}$ is an integer, thus $p+1 \mid n+1$, which gives us $p=n$.
This post has been edited 6 times. Last edited by ZNatox, Jan 5, 2023, 6:46 PM
Z K Y
The post below has been deleted. Click to close.
This post has been deleted. Click here to see post.
X.Allaberdiyev
104 posts
#28
Y by
Interesting. Suppose that $n$ and $p$ are not equal and $p>2$:
We know that $n^p$>$p^n$, since $n,p>e$ ($n=2$ case is impossible because $n$ must be odd according to mod 2 and $n=1$ is easy) we have $p>n$ (It is well known that for $a,b>e$, if $a^b>b^a$, then $b>a$). We have $2 \mid p^n+1\mid n^p+1$, then $n$ is odd, then $p+1 \mid p^n+1 \mid n^p+1$ . Let $\operatorname{ord}_{p+1}(n)=k$, since $n^{2p} \equiv 1 \pmod{p+1}$, we know that $k \mid 2p$, we know that $k<2p$, and $k$ is not equal to $p$ (because $n^p \equiv -1 \pmod{p+1}$), which means that $k \mid 2$, then $n^p+1 \equiv n+1 \pmod{p+1}$, since $p+1 \mid n^p+1$, we have $n \equiv p \pmod{p+1}$, which is contradicting $p>n$, then $p$ must be equal to $n$ for $p,n>3$. For $p=2$ we have $2^n+1>n^2+1$ for $n>4$, you can check the cases $n=1,2,3,4$, which gives the solutions $(2,4)$ and $(2,2)$. Then answers are $(p,p)$ and $(2,4)$ where $p$ is an any prime number.
This post has been edited 4 times. Last edited by X.Allaberdiyev, Aug 17, 2023, 4:43 PM
Z K Y
The post below has been deleted. Click to close.
This post has been deleted. Click here to see post.
Inconsistent
1455 posts
#29
Y by
I claim the answers are $n = p$ for all $p$ and $n = 4, p = 2$.

First, it is well known that $a^b > b^a$ when $a < b$ and both $a, b \geq 3$ (the result follows form taking a log of both sides and a derivative to show strict monotonicity).

In the case when $p = 2$, by bounding we have $n \leq 4$ and here only $n = 4$ works. In the case $n = 1$, the value is less than $1$, and in the case $n = 2$, there is death by parity for $p \geq 3$. Otherwise, we have $n \geq 3$ and thus $n \leq p$. Obviously $n = p$ works.

Now notice that since $p^n + 1$ is even, we must have $n$ odd, so it follows $n^p \equiv -1 \pmod{p + 1}$. In particular, since $p+1$ is composite and thus $p \nmid \phi(p+1)$, it follows that there is a unique solution to $n^p \equiv -1 \pmod{p+1}$ up to $p + 1$. Since $n = p$ suffices, this must be the only solution, so we are done.
Z K Y
The post below has been deleted. Click to close.
This post has been deleted. Click here to see post.
john0512
4188 posts
#30
Y by
The answer is $n=p$ and $p=2,n=4$ only.

For $p=2$, we have $2^n+1\mid n^2+1$, which only has $n=2$ and $n=4$ as a solution since $2^n>n^2$ for all $n\geq 5$. $n=1$ gives $p+1\mid 2$ which clearly does not work, and $n=2$ gives $p^2+1\mid 2^p+1$ which can only work for $p=2$ since $2^p+1$ is odd.

Thus, assume $n,p\geq 3$ from now on. This means that for $n^p$ and $p^n$, the one with the higher exponent is larger, which means that we must have $n\leq p$.

If $n$ is even, we must have $p=2$ since the numerator is odd, but we have already shown that the only solution for $p=2$ is $n=2$ and $n=4$. From now on, assume $n$ odd, so $p+1\mid p^n+1$. Thus, $$p+1\mid n^p+1$$$$n^p\equiv -1\pmod{p+1}$$$$n^{2p}\equiv 1\pmod{p+1}.$$Thus the order of $n$ mod $p+1$ must divide $2p$ but not $p$, which means it is either $2$ or $2p$. However, it cannot be $2p$ since that is larger than the modulus of $p+1$, hence the order is $2$ so $n\equiv -1\pmod{p+1}$. Since $n\leq p$, the only solution is $n=p$ in this case.
Z K Y
The post below has been deleted. Click to close.
This post has been deleted. Click here to see post.
kuzeyaloglu
16 posts
#31
Y by
Only $n = p, (2,4)$ satisfy the condition.
For $p=2, p=3$ we can just check until the denominator gets bigger than the numerator.
So we can assume $p$ is odd and if $n > p \iff p^n > n^p $ so $n \le p$ It is clear that if we
let $\frac{n^p + 1}{p^n + 1} = k$ for some positive integer we have for $p > 2$ $k=1$ if and only if $n = p$ so we can assume $n < p $
$$n^p + 1 - k = p^nk \Rightarrow n \equiv k-1 \pmod p \Rightarrow n = k-1 $$$$(k-1)^p + 1 - k = p^nk \Rightarrow p = k-1 $$so $n = p $ and there are no solutions for $k > 1$.
Z K Y
The post below has been deleted. Click to close.
This post has been deleted. Click here to see post.
mathfun07
38 posts
#32
Y by
Solution pairs $(n,p)$, are $(p,p)$ for all primes, and $(4,2)$. Check $p=2$ manually, and by size, assume that $n<p$, as $n=p$ obviously works. Now assume for sake of contradiction some other $n$ works for an odd prime $p$.

By orders, if a prime $q \mid n^p + 1$, either $q \equiv 1 \pmod p$ or $q\mid n+1$. By parity, $n=p-1$ dies so consider $n<p-1$: this means $n+1 < p$. By LTE, any prime dividing $n+1$ cannot divide $\frac{n^p+1}{n+1}$, but $n^p+1 \equiv n+1 \pmod p$, which means the primes dividing $n^p+1$ that aren't $1 \pmod p$, are exclusively those dividing $n+1$: their product is thus less than $p$. However, as $2 \mid p^n+1$, then there exists some primes that aren't $1 \pmod p$ that have product greater than $1$ which is a contradiction.

I read the other solutions and noticed that I could have just considered $p+1$ instead.
This post has been edited 2 times. Last edited by mathfun07, Mar 26, 2024, 11:04 PM
Z K Y
The post below has been deleted. Click to close.
This post has been deleted. Click here to see post.
alexanderhamilton124
397 posts
#33 • 3 Y
Y by ubermensch, L13832, Nobitasolvesproblems1979
For $p = 2$: We have $2^n > n^2$ for $n \geq 5$, which means we have to check $n = 1, 2, 3, 4$ which gives $(4, 2)$ as a solution, and $(2, 2)$.

From now on, assume $p$ odd. Clearly, $p^n + 1$ is even, which means $n$ has to be odd. Consider a prime $q$ such that $q \mid p + 1$. Since $p + 1 \mid p^n + 1$, $q \mid n^p + 1 \implies n^p \equiv -1 \mod{q} \implies n^{2p} \equiv 1\mod{q}$. So, $\text{ord}_q(n) \mid 2p$, and it can't be $1$ or $p$ since $\text{ord}_q(n) \nmid p$, and it can't be $2p$, since $2p > p + 1 > q - 1$. So, $\text{ord}_q(n) = 2$, and $n \equiv -1 \mod{q}$.

By LTE, we have $v_q(p^n + 1) = v_q(p + 1) + v_q(n)$, and since $q \mid n + 1$, $q \nmid n$, and $v_q(p^n + 1) = v_q(p + 1)$. We have $v_q(n^p + 1) = v_q(n + 1)$, so $v_q(n + 1) \geq v_q(p + 1) \implies p + 1 \mid n + 1$, so $p \leq n$. For $n > p$, however, $p^n > n^p$, a contradiction, which means $n = p$ is the only solution here, and we're done.
Z K Y
The post below has been deleted. Click to close.
This post has been deleted. Click here to see post.
EVKV
71 posts
#34 • 1 Y
Y by Nobitasolvesproblems1979
CASE 1: n=pk+r where $k \geq 4$ and $0 \leq r $< $p$

Claim 1.1 : $p^n$ > $n^p$

Proof 1.1 : This is equivalent to proving $p^{\frac{n}{p}}$ > $n$ => $p^{k}$ > $pk+r$
=> $p^{k}$ > $p(k+1)$ => $p^{k-1}$ > $(k+1)$ which is clearly true for all primes as $k \geq 4$

CASE 2: n=p+r where $0 \leq r $< $p$ $p \neq 2$

Claim 2.1 : Only n=p satisfies this

Proof 2.1 : $p+1 | (p+r)^{p} +1$ so, $(p+r)^{2p} \equiv 1$ mod $p+1$
Now the order of $p+r $mod $p+1$ $\in$ {$2,2p$}
however 2p>p+1 ,thus order is 2
So,$ p+r \equiv -1 $mod $p+1 $ $ p+r \equiv -1 + r \equiv -1$mod $p+1 $
$ r \equiv 0 $mod $p+1 $ thus r=0

Thus n=p

CASE 3: n=p-r where $0 \leq r $< $p$ $p \neq  2$

Claim 3.1 : Only n=p satisfies this

Proof 3.1 : $p+1 | (p-r)^{p} +1$ so, $(p-r)^{2p} \equiv 1$ mod $p+1$
Now the order of $p-r $mod $p+1$ $\in$ {$2,2p$}
however 2p>p+1 ,thus order is 2
So,$ p-r \equiv -1 $mod $p+1 $ $ p-r \equiv -1 + r \equiv -1$mod $p+1 $
$ -r \equiv 0 $mod $p+1 $ thus r=0

Thus n=p

CASE 4: n=2p+r where $0 \leq r $< $p$ $p \neq 2$

Claim 4.1 : This has no solution

Proof 4.1 : $p+1 | (2p+r)^{p} +1$ so, $(2p+r)^{2p} \equiv 1$ mod $p+1$
Now the order of $2p+r $mod $p+1$ $\in$ {$2,2p$}
however 2p>p+1 ,thus order is 2
So,$ 2p+r \equiv -1 $mod $p+1 $ $ 2p+r \equiv -2 + r \equiv -1$mod $p+1 $
$ r \equiv 1 $mod $p+1 $ thus r=1

Claim 4.1.1 : $p^{2p+1}$ > $(2p+1)^p$

Proof 4.1.1 : This is equivalent to proving $p^{\frac{2p+1}{p}}$ > $2p+1$
=> $p^{2}$ > $2p+1$ which is clearly true for all primes as $p \geq 3$

Thus No solution

CASE 5: n=3p+r where $0 \leq r $< $p$ $p\neq2$

Claim 5.1 : This has no solution

Proof 5.1 : $p+1 | (3p+r)^{p} +1$ so, $(3p+r)^{2p} \equiv 1$ mod $p+1$
Now the order of $3p+r $mod $p+1$ $\in$ {$2,2p$}
however 2p>p+1 ,thus order is 2
So,$ 3p+r \equiv -1 $mod $p+1 $ $ 3p+r \equiv -3 + r \equiv -1$mod $p+1 $
$ r \equiv 2 $mod $p+1 $ thus r=2

Claim 5.1.1 : $p^{3p+2}$ > $(3p+2)^p$

Proof 5.1.1 : This is equivalent to proving $p^{\frac{3p+2}{p}}$ > $3p+2$
=> $p^{3}$ > $3p+2$ which is clearly true for all primes as $p \geq 3$

Thus No solution

CASE 6: p=2 n=2k+r

Claim 6.1 : (n,p)=(2,2) , (4,2) satisfy it

Proof 6.1 : By Case 1 $k \leq 3$ which then bashing with both r=1,0 we get only 2,2 and 4,2 satisfy the problem

Done


Remark: Grt problem on bounding and simple orders

I rate it 10 mohs

Solved: 17/4/25
This post has been edited 3 times. Last edited by EVKV, Apr 23, 2025, 6:35 PM
Z K Y
N Quick Reply
G
H
=
a